LSAT and Law School Admissions Forum

Get expert LSAT preparation and law school admissions advice from PowerScore Test Preparation.

 Administrator
PowerScore Staff
  • PowerScore Staff
  • Posts: 8917
  • Joined: Feb 02, 2011
|
#41668
Complete Question Explanation
(The complete setup for this game can be found here: lsat/viewtopic.php?t=4937)

The correct answer choice is (B)

Answer choice (A) is incorrect because it violates the third rule. Answer choice (C) is incorrect because it K cannot work on Wednesday (O is already there). Answer choice (D) is incorrect because it violates the last rule. Answer choice (E) is incorrect because no supervisor would be assigned to Tuesday. Thus, answer choice (B) is correct (and, in answer choice (B), P would also have to work on Monday).
 Sdaoud17
  • Posts: 85
  • Joined: Apr 13, 2013
|
#9308
Can you explain how D is conflicting with last rule . :-? :-?

I mean if

you can have K on monday and Thursday with N. which make K woke first day during the week.

Thank you
User avatar
 Dave Killoran
PowerScore Staff
  • PowerScore Staff
  • Posts: 5852
  • Joined: Mar 25, 2011
|
#9310
Sure! Let's start by taking a closer look at the last rule again, and I'll add italics for emphasis:

"Any day on which Kevin works is the first day during the week that some other staff member works."

So, if Kevin is working on a day, the other person working with him that day is working for the first time that week.

Under answer choice (D), Nan works with Ophelia first on Wednesday, and then with Kevin on Thursday, so Thursday with Kevin won't be her first working day that week, a violation of that last rule.

Please let me know if that helps. Thanks!
 Sdaoud17
  • Posts: 85
  • Joined: Apr 13, 2013
|
#9316
Thank you so much I got it
Very tricky rule , I was rushing :x :x :)
 lolaSur
  • Posts: 46
  • Joined: Nov 11, 2019
|
#72242
For question 22 which states "which one of the following could be true?" there are 4 cannot be true incorrect answers and 1 could be true correct answer right?

After reading the explanation to this question, I understand why Answer B is correct. However, it wasn't immediately clear to me that P could work on Monday and on Tuesday. At first glance, answer B seemed wrong to me because I thought it violated the rule that S cannot appear before P.

Is it possible to figure out a ratio for this game? If yes, could you please tell me what is the ratio?


Thank you so much in advance!

For my reference (L4- game #5)
Last edited by lolaSur on Sat Feb 01, 2020 10:00 pm, edited 1 time in total.
User avatar
 Stephanie Oswalt
PowerScore Staff
  • PowerScore Staff
  • Posts: 811
  • Joined: Jan 11, 2016
|
#72256
Hi Lola,

I moved your posts to the thread discussing #22 (you posted in the "setup" section — we actually offer an overall setup and an explanation of each question :D ). Please review the above explanations, and let us know if that helps! Thanks! :)

Get the most out of your LSAT Prep Plus subscription.

Analyze and track your performance with our Testing and Analytics Package.